0 Daumen
268 Aufrufe

Aufgabe:

Seien \( n \in \mathbb{N}, a_{0}, \ldots, a_{n-1} \in \mathbb{R} \) und \( P: \mathbb{R} \rightarrow \mathbb{R}, P(x):=x^{n}+a_{n-1} x^{n-1}+\ldots+a_{1} x+a_{0} \) ein Polynom von Grad \( n \).

(i) Zeigen Sie, dass \( P \) nicht beschränkt ist.

(ii) Sie nun \( n=2 k \) für ein \( k \in \mathbb{N} \). Zeigen Sie, dass \( P \) nach unten beschränkt ist.


Problem/Ansatz:

Leider finde ich keinen Ansatz, wie ich an die Aufgabe heran gehen soll. Ich weiss nicht wie ich beweisen soll, dass es nicht beschränkt ist. Bitte um Eure Hilfe.

Vielen Dank im Voraus

Avatar von

1 Antwort

0 Daumen

 \( P(x):=x^{n}+a_{n-1} x^{n-1}+\ldots+a_{1} x+a_{0} \)

Schreibe \(P\) in der Form \(x^n\cdot (1+a_{n-1}\cdot \frac{1}{x}+\cdots+a_0\cdot \frac{1}{x^n})\).

Was geschieht mit dem Klammerfaktor, wenn \(x\to\pm \infty\) ?

Avatar von 29 k

Ein anderes Problem?

Stell deine Frage

Willkommen bei der Mathelounge! Stell deine Frage einfach und kostenlos

x
Made by a lovely community